LSAT and Law School Admissions Forum

Get expert LSAT preparation and law school admissions advice from PowerScore Test Preparation.

 srcline@noctrl.edu
  • Posts: 243
  • Joined: Oct 16, 2015
|
#33199
Hello

So I'm not under standing why D is the correct answer here. I understand that there is a conditional in the stimulus and diagrammed it as:

antenna to work well :arrow: Symmetrical and Fractal structure
contra (+): not sysmmetrical OR not fractal structure :arrow: attenna will not work.

So that's essentially what the conclusion is saying right? D does not make any sense to me.....the antenna has to be both symmetrical and have a fractal structure ....these are necessary conditions, which the authors states that they have......

Can someone please explain this question.

Thanks
Sarah
User avatar
 Jonathan Evans
PowerScore Staff
  • PowerScore Staff
  • Posts: 726
  • Joined: Jun 09, 2016
|
#33235
Hi, Sarah,

Good question. Good diagramming. Just make sure you understand that there's slightly more that what you have on these diagrams, that your diagram leaves implicit a couple points. To wit:

Antenna to work equally well at all frequencies :arrow: symmetrical & fractal

symmetrical OR fractal :arrow: Antenna to work equally well at all frequencies

Obviously you don't want to be unnecessarily prolix in your diagramming, so you might condense to:

AW :arrow: S & F
S OR F :arrow: AW

So far so good. Now, any other premises here? What do we know about the particular antenna in question? It does NOT "work equally well at all frequencies."

this antenna :arrow: AW

What does the author conclude from AW? He concludes that it is not the case that:

Antenna to work equally well at all frequencies :arrow: symmetrical & fractal

In other words, the author believes that AW is sufficient to know that S OR F. Now, see whether you can spot the issue.

The author has made a Mistaken Negation™:

AW :arrow: S OR F

Note that this Mistaken Negation™ is the contrapositive of the Mistaken Reversal™:

S & F :arrow: AW

Answer Choice (D) does describe this faulty reasoning:

The author "interprets an assertion that certain conditions are necessary as asserting that those conditions are sufficient."

Note from a strategy point of view, you could rule out all but (C) and (D) because only these two are related to conditionals. If you are able at least to note a problem with mistaken negation or reversal, you could establish that (D) is the correct answer.

Does this help? Thanks!
 srcline@noctrl.edu
  • Posts: 243
  • Joined: Oct 16, 2015
|
#33263
Hello Jonathan

Okay, I think I see this now...I left out the other diagram, while I was taking my test I had automatically ruled out A, B, C so that saved me some time there....

SO the big picture idea is that that the author is treating symmetrical and fractal structure as sufficient conditions even though they are both necessary conditions for an antenna to work.

So D is saying that the author treats these two elements as sufficient even though in both instances they are necessary for an antenna to work well.

Thankyou for explaining this, makes a lot more sense now.
Sarah
 hassan66
  • Posts: 51
  • Joined: Jul 19, 2018
|
#49801
Hi, I initially got this wrong because I didn't see the conditional logic in it and didn't diagram. So to clarify what has been discussed above, we are told A--> B + C; we are then told that we have B + C (the antenna is symmetrical and has a fractal structure), and based on the fact that we have B+C but not A (the antenna working equally well at all frequencies), the expert claims that the claim is incorrect. But from basic conditional logic we only know that A yields B+C but not that if we have B+C that must mean that we had A. Is it correct to say that in the group of antennas that are symmetrical in shape and have a fractal structure, we only know that some of those antennas work equally well at all frequencies but there could be antennas that satisfy both those conditions but that are just faulty? Do we even need to diagram the second part when by looking at A--> B + C we know that satisfying the necessary condition does not mean we had the sufficient condition.

Thank you!
 Adam Tyson
PowerScore Staff
  • PowerScore Staff
  • Posts: 5153
  • Joined: Apr 14, 2011
|
#50064
You got it, hasan66! Well done! Satisfying the necessary conditions tells us nothing about whether the sufficient conditions have been satisfied, and that is were the error lies. Nice work, good analysis!
 hassan66
  • Posts: 51
  • Joined: Jul 19, 2018
|
#57067
Thank you! :)
User avatar
 schocktherapy
  • Posts: 5
  • Joined: Dec 16, 2023
|
#104711
Hi guys,

Happy New Year 2024!

Would this be the same as saying:

Researchers have discovered that in order to execute somebody, an individual must fire a black revolver from exactly 15 yards away from the victim/decedent. However, weeks later, employees hired by the R&D firm that published the study, acquired a black revolver instead exactly 15 yd away from another woman and fired at the unsuspecting woman. Because the woman lived, we can safely conclude that the researchers claims are false.

Sorry for the violent nature of the hypothetical...lol. It was all I could think of at the time.. lol

Nobody was hurt during my cogitations..

But is the key issue weather the gun and the distance (in yards) necessary or sufficient?


Thank you so much guys! 8-)
User avatar
 srusty
PowerScore Staff
  • PowerScore Staff
  • Posts: 32
  • Joined: Nov 30, 2023
|
#104896
Hi Schock Therapy! Your example makes sense, yes, in the way that we don’t know what other factors could affect the … execution. We know that *at minimum* we need the distance and instrument. In our stimulus, we know we need AT LEAST symmetry and fractal structure. The other considerations that guarantee an antenna working well at all frequencies are not provided. So, we know that these two qualities are necessary, but we can’t state that they will be sufficient in making an antenna that works well at all frequencies. Hope this helps!

Get the most out of your LSAT Prep Plus subscription.

Analyze and track your performance with our Testing and Analytics Package.